Đến nội dung

IloveMaths nội dung

Có 162 mục bởi IloveMaths (Tìm giới hạn từ 13-05-2020)



Sắp theo                Sắp xếp  

#446484 cho hai đường tròn tiếp xúc nhau, chứng minh góc bằng nhau

Đã gửi bởi IloveMaths on 31-08-2013 - 09:18 trong Hình học phẳng

Cho tam giác ABC nội tiếp đường tròn tâm O,đường tròn tâm $O_{1}$ tiếp xúc với AB, AC tại P, Q và tiếp xúc với (O) tại S, AS cắt PQ tại D . Cm:$\angle BDP=\angle CDQ$  :botay 

:icon6: giải như sau :

Dễ dàng chứng minh SP,SQ lần lượt là phân giác các góc ASB và góc ASC

Xét tam giác BPD và tam giác CQD có:

$\widehat{BPD}=\widehat{CQD}$

Ta cần chứng minh $\frac{BP}{CQ}=\frac{PD}{QD}$

Theo tính chất đường phân giác suy ra

$\frac{SB}{SC}=\frac{BP}{QC}$

Mà $\frac{SB}{SC}=\frac{sin \widehat{SCB}}{sin\widehat{SBC}}=\frac{sin \widehat{BAS}}{sin\widehat{SAC}}$

Việc còn lại  cần chứng minh :

$\frac{PD}{QD}=\frac{sin\widehat{SAB}}{sin\widehat{SAC}}\Leftrightarrow \frac{PD}{sin\widehat{SAB}}=\frac{QD}{sin\widehat{SAC}}(=\frac{AD}{sin\widehat{APQ}}=\frac{AD}{sin\widehat{AQP}})$

$\Rightarrow Q.E.D$ :icon6:  :lol: 




#446041 Cuộc thi giải toán vectơ bằng nhiều cách (29/08 - 08/09)

Đã gửi bởi IloveMaths on 29-08-2013 - 06:49 trong Hình học phẳng

:icon6:  :icon6:  :icon6:  :icon6: hay quá 




#439519 đề ra kì này THTT số 433

Đã gửi bởi IloveMaths on 31-07-2013 - 19:45 trong Toán học & Tuổi trẻ

Có ai có đề ra kì này THTT số 433 post lên cho mình vơi.Cảm ơn . :icon6: 




#432710 Cho $x+y=1$. Tìm GTLN của $\frac{x^3}{x+1...

Đã gửi bởi IloveMaths on 04-07-2013 - 10:03 trong Bất đẳng thức và cực trị

Cho $x,y$ là các số thực không âm thỏa mãn $x+y=1$. Tìm giá trị lớn nhất của biểu thức:

$$P=\frac{x^3}{x+1}+\frac{y^3}{y+1}$$

Áp dụng bất đẳng thức Cauchy:

$\frac{x^3}{x+1}+\frac{x+1}{4}+\frac{1}{2}+\frac{x}{2}\geq x$

$\frac{y^3}{y+1}+\frac{y+1}{4}+\frac{1}{2}+\frac{y}{2}\geq y$

$\Rightarrow \frac{x^3}{x+1}+\frac{y^3}{y+1}\geq \frac{3}{4}$




#426722 cho tg ABCD noi tiep co AC va BD vuong goc .Lay M tren AB sao cho AM=1/3 AB ....

Đã gửi bởi IloveMaths on 13-06-2013 - 11:37 trong Hình học phẳng

cho tg ABCD noi tiep co AC va BD vuong goc .Lay M tren AB sao cho AM=1/3 AB . N trung diem HC.

CMR: DN vuong goc voi MH

:icon6:  :icon6:  Sử dụng véc tơ  là xong luôn  :lol: 




#426603 cho tg ABCD noi tiep co AC va BD vuong goc .Lay M tren AB sao cho AM=1/3 AB ....

Đã gửi bởi IloveMaths on 12-06-2013 - 22:40 trong Hình học phẳng

cho tg ABCD noi tiep co AC va BD vuong goc .Lay M tren AB sao cho AM=1/3 AB . N trung diem HC.

CMR: DN vuong goc voi MH

:icon6:  :icon6:  :icon6:  H là điểm nào vậy




#425063 $\frac{MA}{BC}+\frac{MB}{AC...

Đã gửi bởi IloveMaths on 08-06-2013 - 15:56 trong Hình học

Với mọi M thuộc tam giác ABC chứng minh rằng: $\frac{MA}{BC}+\frac{MB}{AC}+\frac{MC}{AB}\geq \frac{AB+AC+BC}{MA+MB+MC}$

:icon6:  Xem lời giải THTT T12/419 




#424806 Cho đường tròn (O) và ba dây cung AB,CD,EF thỏa mãn : AB,CD,EF đồng quy tại I...

Đã gửi bởi IloveMaths on 07-06-2013 - 16:59 trong Hình học phẳng

Cho đường tròn (O) và ba dây cung AB,CD,EF thỏa mãn : AB,CD,EF đồng quy tại I và AC,BD,EF đồng quy tại J. Gọi

 $M=AD\cap BC$

$N=AE\cap BF$

Chứng minh M,N,J thẳng hàng  :icon6:  :icon6:  :icon6:  :icon6: 




#424015 Cho tam giác ABC. Đường tròn (O) cắt BC tại X,Y ; cắt AC tại Z,T ; cắt AB tại...

Đã gửi bởi IloveMaths on 05-06-2013 - 09:24 trong Hình học phẳng

tớ vẽ hoài mak M P N có thẳng hàng đâu

:icon6:  :icon6:  :icon6:  :icon6:  Đề ko sai đâu  :icon6:  :icon6:  :icon6: 




#423731 Một số bài hệ phương trình chứa căn thức

Đã gửi bởi IloveMaths on 04-06-2013 - 14:11 trong Phương trình - hệ phương trình - bất phương trình

11.$\left\{\begin{matrix} \sqrt{x}+\sqrt[4]{32-x}-y^{2}=-3\\ \sqrt[4]{x}+\sqrt{32-x}+6y=-24 \end{matrix}\right.$

:icon6: Xem tại đây 

http://diendantoanho.../97686-giải-hệ/




#423702 PT-HPT-BPT Tuyển tập các bài toán sưu tầm từ Mathslink.ro

Đã gửi bởi IloveMaths on 04-06-2013 - 12:01 trong Phương trình - Hệ phương trình - Bất phương trình



Bài toán 39: Tìm tất cả các nghiệm của PT $ (a^{2}+2b^{2}+2ab)(a^{2}+2b^{2}-2ab)=3b^{2}(2(a^{2}-b^{2})^{2}(a^{2}+b^{2}))^{\frac{1}{3}} $

 

 

Ta có:

$VP=(a^2+2b^2+2ab)(a^2+2b^2-2ab)=((a+b)^2+b^2)((a-b)^2+b^2)=((a^2-b^2)^2)+b^4+b^2.2.(a^2+b^2)\geq 3b^2.\sqrt[3]{2.(a^2-b^2)^2.(a^2+b^2))}=VT$

Dấu đẳng thức xảy ra khi 

$(a^2-b^2)^2=2b^2(a^2+b^2)=b^4$

-Nếu $b=0 \Rightarrow a=0$ thỏa mãn

-Nếu $b\neq 0\Rightarrow 2a^2+2b^2=b^2\Rightarrow 2a^2=-b^2$ (vô lí)

Vậy nghiêm của phương trình là:

(a;b)=(0;0)

:icon6:  :icon6:  :icon6:




#423666 Cho tam giác ABC. Đường tròn (O) cắt BC tại X,Y ; cắt AC tại Z,T ; cắt AB tại...

Đã gửi bởi IloveMaths on 04-06-2013 - 10:14 trong Hình học phẳng

Cho tam giác ABC. Đường tròn (O) cắt BC tại X,Y ; cắt AC tại Z,T ; cắt AB tại U,V sao cho XYZTUV là một lục giác. 

Gọi $UZ \cap TY=B' ; VY\cap XZ=A'; VT\cap UX=C'$

Gọi $AB \cap A'B'=M ; BC\cap B'C'=P; AC\cap A'C'=N$

 

Chứng minh :

a) M,P,N thẳng hàng 

b) Các đường thẳng TX,ZV,UY lần lượt đi qua M,P,N


 




#423413 Giải hệ phương trình: $\left\{\begin{matrix...

Đã gửi bởi IloveMaths on 03-06-2013 - 10:52 trong Phương trình - hệ phương trình - bất phương trình

làm thì phải làm đến ra kq chứ tớ cần đối chiếu mak

Thì thay lần luot $\frac{x}{y}=\frac{-1}{2};-2;-1$ vào $x^2+xy+y^2=3$ là xong  :luoi:  :luoi: 




#423409 Giải hệ phương trình: $\left\{\begin{matrix...

Đã gửi bởi IloveMaths on 03-06-2013 - 10:45 trong Phương trình - hệ phương trình - bất phương trình

Ta có:

$7(x^5+y^5).3=31(x^3+y^3).(x^2+xy+y^2)\Leftrightarrow 10x^5+10y^5+31(x^4y+x^3y^2+y^3x^2+xy^4)=0(1)$

Xét hai truong hợp:

TH1 : y =0 , thử thấy ko thỏa mãn 

TH2 : $y\neq 0$

Chia hai vế của (1) cho $y^5$ 

$(1)\Leftrightarrow 10.(\frac{x}{y})^5+10+31.((\frac{x}{y})^4+(\frac{x}{y})^3+(\frac{x}{y})^2+(\frac{x}{y}))$

Đăt $\frac{x}{y}=t$

$(1)\Leftrightarrow 10t^5+10+31(t^4+t^3+t^2+t)=0\Leftrightarrow 5t^4(2t+1)+13t^3.(2t+1)+9t^2(2t+1)+11t(2t+1)+10(2t+1)=0\Leftrightarrow (2t+1)(5t^4+13t^3+9t^2+11t+10)=0\Rightarrow (2t+1)(t+1)(5t^3+8t^2+t+11)=(2t+1)(t+1)(t+2)(5t^2-2t+5)=0$

Đến đây Huyenpluss tu giải tiếp =))  :icon6:  :icon6:  :icon6:  :icon6:  :icon6: 

 




#421431 $x_{1}+x_{2}+...+x_{n+1}=x_{1}.x...

Đã gửi bởi IloveMaths on 27-05-2013 - 11:04 trong Dãy số - Giới hạn

Cho dãy số xác định bởi:

$x_{1}=2013$

$x_{1}+x_{2}+...+x_{n+1}=x_{1}.x_{2}...x_{n+1}$$\forall n\geq 1$

Chứng minh day số có giới hạn ,tìm giới hạn đó 




#420908 $\left\{ \begin{array}{l} 4x^2+3...

Đã gửi bởi IloveMaths on 25-05-2013 - 09:16 trong Phương trình - hệ phương trình - bất phương trình

hpt$\Leftrightarrow \left\{\begin{matrix} (x-1)(4x+3y+4)=0)\\ 3y^{2}+4x(y-1)=3 \end{matrix}\right.$

$\Leftrightarrow \left\{\begin{matrix} x-1=0\\ 3y^{2}+4x(y-1)=3 \end{matrix}\right.$

hoặc

$\left\{\begin{matrix} 4x+3y+4=0\\ 3y^{2}+4x(y-1)=3 \end{matrix}\right.$

$\Rightarrow$ hệ pt có 3 nghiệm (x;y)=(1;1);(1;$\frac{-7}{3}$);($\frac{-7}{4}$;1)

:luoi:  :luoi:  :luoi:  :luoi:  :luoi:  :luoi:  :luoi:  :luoi:  :luoi:  :luoi:  :luoi:  :luoi:  :luoi:  :luoi:

 

Bài làm rat tot, ý tuong như tớ ...... :icon6:  :icon6:  :icon6: có ngắn hơn thì phải.......... :luoi: 




#420796 Đề tuyển sinh THPT chuyên Lam Sơn (Sở GD và ĐT Thanh Hóa)

Đã gửi bởi IloveMaths on 24-05-2013 - 21:33 trong Tài liệu - Đề thi

Câu 3: (2,0 điểm)

2. Tìm tất cả các số nguyên tố $p$ để $4p^{2}+1$ và $6p^{2}+1$ cũng là số nguyên tố.

 

Xét so dư của p khi chia cho 5 , ta có các so du là 0,1,2,3,4

- Nếu $p=5k+1(k\geq 1)\Rightarrow 4p^2+1=4.(5k+1)^2+1\vdots 5$ và $p>5\Rightarrow$ không là số nguyên tô

- Néu $p=5k+2(k\geq 1)\Rightarrow 6p^2+1=6(5k+2)^2+1\vdots 5$ và $p>5\Rightarrow$ không là sô nguyên tố

-Nếu $p=5k+3(k\geqslant 1)\Rightarrow 6p^2+1=6.(5k+3)^2+1\vdots 5$ và $p>5\Rightarrow$ không là số nguyên tô

-Nếu $p=5k+4\Rightarrow 4p^2+1=4.(5k+4)^2+1\vdots 5$ và $p>5\Rightarrow$ không là số nguyên tô

Vậy p = 5




#420765 Đề tuyển sinh THPT chuyên Lam Sơn (Sở GD và ĐT Thanh Hóa)

Đã gửi bởi IloveMaths on 24-05-2013 - 20:29 trong Tài liệu - Đề thi

dự đoán Max = 3 mình dùng tam thức bậc 2 hả ?  :icon1:

:icon6:  :icon6:  :icon6:  :icon6: 

Thử vài giá trị thôi .............. :lol: 




#420718 Đề tuyển sinh THPT chuyên Lam Sơn (Sở GD và ĐT Thanh Hóa)

Đã gửi bởi IloveMaths on 24-05-2013 - 17:30 trong Tài liệu - Đề thi

 

Câu 2: (2,0 điểm) Cho phương trình: $ax^{2}+bx+c=0 (a\neq 0)$ có hai nghiệm $x_{1},x_{2}$ thỏa mãn điều kiện: $0\leq x_{1}\leq x_{2}\leq 2$. Tính GTLN của biểu thức:

                             $Q=\frac{2a^{2}-3ab+b^{2}}{2a^{2}-ab+ac}$

 

 

Theo Vi-ét:  $x_{1}+x_{2}=\frac{-b}{a};x_{1}x_{2}=\frac{c}{a}$

Ta có:

$Q=\frac{2a^{2}-3ab+b^{2}}{2a^{2}-ab+ac}$=$\frac{{2-\frac{3.b}{a}+(\frac{b}{a})^2}}{2-\frac{b}{a}+\frac{c}{a}}$=$\frac{2+3(x_{1}+x_{2})+(x_{1}+x_{2})^2}{2+(x_{1}+x_{2})+x_{1}.x_{2}}$

Dự đoán Max = 3

Do đó ta cần chứng minh:

$2+3(x_{1}+x_{2})+(x_{1}+x_{2})^2\leq 6+3(x_{1}+x_{2})+3.x_{1}.x_{2}\Leftrightarrow x_{1}^2+x_{2}^2-x_{1}.x_{2}\leq 4$

Xét hai truong hop:

-Nếu $x_{1}=0\Rightarrow x_{2}^2\leq 4$ ( đúng do $x_{2}\leq 2$ ) 

-Nếu $x_{1}\neq 0\Rightarrow x_{1}^2+x_{2}^2-x_{1}.x_{2}\leq x_{1}^2+x_{2}^2-x_{1}.x_{1}= x_{2}^2\leq 4$ ( đúng do $x_{2}\leq 2$ )

 

Vậy Max = 3 . Dấu bằng xảy ra khi $(x_{1};x_{2})=(0;2),(2;2)$

 

:luoi:  :luoi:  :luoi:  :icon6:  :icon6:  :icon6:  :icon6:  :icon6:  :icon6:  :icon6:  :icon6:  :icon6:  :icon6:  :icon6:  :icon6:  :icon6:  :icon6:  :icon6:  :icon6:  :icon6:  :icon6:  :icon6:  :icon6:  :icon6:




#420568 Poland NMO 2013

Đã gửi bởi IloveMaths on 23-05-2013 - 21:23 trong Thi HSG Quốc gia và Quốc tế

Câu 5. Cho $k,m,n$ là ba số dương phân biệt. Chứng minh rằng:

$$\left( {k - \frac{1}{k}} \right)\left( {m - \frac{1}{m}} \right)\left( {n - \frac{1}{n}} \right) \le kmn - (k + m + n)$$

 

Đề sai mà............. 

 

Cho $k=\frac{1}{2};m=\frac{1}{3};n=25$

 

 




#420506 giải hệ

Đã gửi bởi IloveMaths on 23-05-2013 - 17:21 trong Phương trình - Hệ phương trình - Bất phương trình



giải hệ

 

 

 

$\left\{\begin{matrix} &\sqrt{x} &+ \sqrt[4]{32-x} =-3+y^{2}\\ & \sqrt[4]{x} + & \sqrt{32-x}=24-6y \end{matrix}\right.$

Ta có:

$\sqrt{x}+\sqrt{32-x}+\sqrt[4]{x}+\sqrt[4]{32-x}=y^2-6y+21$

Áp dụng bât đẳng thức bu nhi a cop xki đuoc:

$\sqrt{x}+\sqrt{32-x}\leq 8$

$\sqrt[4]{x}+\sqrt[4]{32-x}\leq 4$

$\Rightarrow \sqrt{x}+\sqrt{32-x}+\sqrt[4]{x}+\sqrt[4]{32-x}\leq 12\Rightarrow y^2-6y+21\leq 12\Leftrightarrow (y-3)^2\leq 0\Rightarrow y=3$

Vậy nghiêm của hệ là:

$(x;y)=(16;3)$

:luoi: 




#420448 $\left\{ \begin{array}{l} 4x^2+3...

Đã gửi bởi IloveMaths on 23-05-2013 - 10:22 trong Phương trình - hệ phương trình - bất phương trình

Số nghiệm của hệ (1) 4X^2 +3 Y(X-1)=4 vs (2) 3Y^2 + 4X(Y-1)=3 là?

:luoi:  để mình giúp bạn hí

$(1)\Leftrightarrow 4x^2+3y(x-1)=4\Leftrightarrow 4(x-1)(x+1)=-3y(x-1)$

Xét hai truong hop:

-Nếu $x=1$ thay vào (2) ta duoc:

$(2)\Leftrightarrow 3y^2+4.1.(y-1)=3\Rightarrow y=1;y=\frac{-7}{3}$

- Néu x khác 1 thì $4(x+1)=-3y\Rightarrow 4x=-3y-4$ thay vào (2) duoc

$3y^2+(-3y-4)(y-1)=3\Rightarrow -y+4=3\Rightarrow y=1\Rightarrow x=\frac{-7}{4}$

Vậy hệ phương trình có nghiêm:

$(x;y)=(1;1);(1;\frac{-7}{3});(\frac{-7}{4};1)$




#420429 $\frac{1}{a}+\frac{1}{b...

Đã gửi bởi IloveMaths on 23-05-2013 - 09:51 trong Bất đẳng thức và cực trị

$\frac{1}{a}+\frac{1}{b}+\frac{1}{c}\geq \frac{9}{a+b+c}$

 

:luoi:




#420427 $f(m+n)+f(n-m)=f(3n)$

Đã gửi bởi IloveMaths on 23-05-2013 - 09:48 trong Phương trình hàm

1.Cho $f: \mathbb{N}\rightarrow \mathbb{R}$ Tìm hàm thỏa mãn:

$f(m+n)+f(n-m)=f(3n)$  và $n,m \in \mathbb{N}$, $n\geq m$




#420257 cho x, y là các số thực duơng thỏa mãn: $x^2+y^2=13$ Tìm max:...

Đã gửi bởi IloveMaths on 22-05-2013 - 17:10 trong Bất đẳng thức và cực trị

Ta chứng minh Max = 9 .  Thật vậy:

$\frac{x^2+6xy}{y^2-2xy+13}\leq 9\Leftrightarrow \frac{x^2+6xy}{y^2-2xy+x^2+y^2}\leq 9\Leftrightarrow x^2+6xy\leq 18y^2+9x^2-18xy\Leftrightarrow 18y^2+8x^2-24xy\geq 0\Leftrightarrow \frac{18y^2+8x^2-24xy}{x^2}\geqslant 0\Leftrightarrow(\frac{y}{x}-\frac{2}{3})^2\geq 0$